Finding the Kernel and Range of Linear Operators on R3

Click For Summary
The discussion focuses on finding the kernel and range of the linear operator L(X) = (x1, x1, x1)T on R3. The kernel is determined to be the set of vectors of the form (0, x2, x3), indicating that x1 must be zero. This leads to the conclusion that the kernel is a 2-dimensional subspace, as it can be spanned by the basis vectors (0, 1, 0) and (0, 0, 1). The range of the operator consists of vectors where all components are equal, specifically of the form (c, c, c) for some scalar c. Overall, the kernel's dimension is confirmed to be two due to the restriction imposed by x1 being zero.
Mdhiggenz
Messages
324
Reaction score
1

Homework Statement



Determine the kernel/range of each of the following linear operators on R3

L(X)=(x1,x1,x1)T

Homework Equations


The Attempt at a Solution



So first thing I did was create a 3x1 matrix filled with ones.

I equaled it to zero and found x1=0 to be a solution. However I'm not quite sure how they come up with the following answer.

(0,x2,x3). Also why would it be a 2 dimensional subspace? Would it be due to x1 being zero?

Thanks
 
Last edited by a moderator:
Physics news on Phys.org
Mdhiggenz said:

Homework Statement



Determine the kernel/range of each of the following linear operators on R3

L(X)=(x1,x1,x1)T

So first thing I did was create a 3x1 matrix filled with ones.

I equaled it to zero and found x1=0 to be a solution. However I'm not quite sure how they come up with the following answer.

(0,x2,x3). Also why would it be a 2 dimensional subspace? Would it be due to x1 being zero?
x1 = 0 if and only if the vector is of the form (0,x2,x3). The subspace spanned by vectors of this form has dimension 2 because for example {(0,1,0), (0,0, 1)} is a basis.
 
Perfect explanation. Thank you!
 
Question: A clock's minute hand has length 4 and its hour hand has length 3. What is the distance between the tips at the moment when it is increasing most rapidly?(Putnam Exam Question) Answer: Making assumption that both the hands moves at constant angular velocities, the answer is ## \sqrt{7} .## But don't you think this assumption is somewhat doubtful and wrong?

Similar threads

  • · Replies 24 ·
Replies
24
Views
3K
  • · Replies 6 ·
Replies
6
Views
2K
  • · Replies 9 ·
Replies
9
Views
3K
  • · Replies 8 ·
Replies
8
Views
3K
  • · Replies 1 ·
Replies
1
Views
1K
Replies
8
Views
2K
  • · Replies 1 ·
Replies
1
Views
2K
  • · Replies 3 ·
Replies
3
Views
2K
  • · Replies 5 ·
Replies
5
Views
2K
  • · Replies 8 ·
Replies
8
Views
3K